Đến nội dung

Chris yang nội dung

Có 231 mục bởi Chris yang (Tìm giới hạn từ 24-05-2020)



Sắp theo                Sắp xếp  

#612915 x+2y+3z=3

Đã gửi bởi Chris yang on 04-02-2016 - 21:30 trong Bất đẳng thức và cực trị

làm sao bạn có được đánh giá như vậy ?? chỉ mình với cám ơn  :D  :D  :D  :D

Dùng BĐT $a^3+b^3\geq ab(a+b)\Rightarrow \frac{11b^3-a^3}{ab+4b^2}\leq \frac{11b^3-b(a^2+ab-b^2}{ab+4b^2}=\frac{12b^2-a^2-ab}{a+4b}=3b-a$




#622285 Việt Nam TST 2016 - Thảo luận đề thi

Đã gửi bởi Chris yang on 24-03-2016 - 17:38 trong Thi HSG Quốc gia và Quốc tế

Bài 1 có được hiểu là $a^n-1$ và $a^{3^{2016}}-1$ có cùng tập ước nguyên tố không nhỉ? Mình không chắc chắn nhưng nếu là vậy thì định lý Zsigmondy có thể giải quyết nó.




#563829 Tìm x,y nguyên thỏa mãn $1+x+x^2+x^3=2^y$

Đã gửi bởi Chris yang on 05-06-2015 - 22:46 trong Số học

Bài giải:

Dễ thấy $x,y$ phải là các sô nguyên không âm

Nếu $x=0$ thì $y=0$

Nếu $x>0$ thì $x$ lẻ

PT tương đương $(x+1)(x^2+1)=2^y$. Khi đó tồn tai $a,b\in\mathbb{N}$ thỏa mãn

$\left\{\begin{matrix} x+1=2^a & \\ x^2+1=2^b & \end{matrix}\right.(a+b=y)$

$\Rightarrow 2x=2^{2a}-2^b\Rightarrow x=2^{2a-1}-2^{b-1}$

Do $x$ lẻ nên dễ dàng suy ra $2a-1$ hoặc $b-1$ bằng $0$. Thu được $b=1\rightarrow x=1\rightarrow y=2$




#617159 Tìm tất cả các số nguyên dương n sao cho: 2n - 1 ⋮ 7

Đã gửi bởi Chris yang on 27-02-2016 - 00:44 trong Số học

Tìm tất cả các số nguyên dương n sao cho: 2n - 1 ⋮ 7

Xét $n=7k,7k+1,7k+2,.....,7k+6$ có $2n-1\vdots 7$ khi mà $2i-1\vdots 7$ với $i=0,1,2,...,6$. Thay vào ta tìm được $i=4$

Vậy những số $n=7k+4$ thì thỏa mãn bài toán =))




#658153 Tìm tất cả các số nguyên dương $(q, r, p),$ với $p$ là số...

Đã gửi bởi Chris yang on 16-10-2016 - 22:02 trong Số học

Tìm tất cả các số nguyên dương $(q, r, p),$ với $p$ là số nguyên tố thỏa mãn $2^{q}+r^{2}=2p.$

Dễ thấy $p$ lẻ

Từ PT suy ra $r$ chẵn $\Rightarrow 4|r^2$. Thấy $2^{q-1}=p-\frac{r^2}{2}$ lẻ do $p$ lẻ nên $q-1=0\Rightarrow q=1$

Khi đó $r^2=2(p-1)\Rightarrow p-1=2^{2k+1}x^2$ với $k,x\in\mathbb{N}$ hay $p$ có dạng $2^{2k+1}x^2+1$ ( Theo nguyên lí Dirichlet về sự tồn tại vô số số nguyên tố dạng $ak+b$ thì $p$ có vô số giá trị thỏa mãn $3,19,73,163,883....$)

 Vậy $(p,q,r)=(2^{2k+1}x^2+1,1,2^{k+1}x)$ với $k,x\in\mathbb{N}$




#587526 Tìm tất cả các cặp số nguyên dương $(a,b)$ thõa mãn: $a^2+b^2...

Đã gửi bởi Chris yang on 05-09-2015 - 23:13 trong Số học

Bài này bữa trước khi chủ topic mới mở mình đã xem qua 1 lần rồi mới biết :D Đâu phải thánh đào :D




#587520 Tìm tất cả các cặp số nguyên dương $(a,b)$ thõa mãn: $a^2+b^2...

Đã gửi bởi Chris yang on 05-09-2015 - 22:59 trong Số học

Tìm tất cả các cặp số nguyên dương $(a,b)$ thõa mãn: $a^2+b^2 = k(ab-1)$

 

Spoiler

Xem thêm ở đây




#587533 Tìm tất cả các cặp số nguyên dương $(a,b)$ thõa mãn: $a^2+b^2...

Đã gửi bởi Chris yang on 06-09-2015 - 00:11 trong Số học

Nhưng ở cái Link đó đã chỉ ra hết nghiệm của phương trình đó đâu, nó chỉ chứng minh có vô số nghiệm thôi :(

PT dạng đó có vô số nghiệm xây dựng theo dãy mà chị Polar ghi đó bạn. Nó chỉ ra vô số nghiệm và cả công thức nghiệm. :)

Thực ra ghi như thế nhưng xây dựng vậy bạn đã chỉ ra hết nghiệm của PT rồi. Nếu muốn khẳng định lại, bạn có thể chọn cặp $(a_i,b_i)$ là nghiệm mà không thuộc dãy trên thỏa mãn $a_i+b_i$ min sẽ thấy điều vô lý




#617151 Tìm tất cả $n$ để $p_{n} \ge (\sqrt{2...

Đã gửi bởi Chris yang on 26-02-2016 - 22:58 trong Số học

Ký hiệu $p_{i}$ là số nguyên tố thứ $i$. Tìm tất cả $n$ để $p_{n} \ge (\sqrt{2})^{n}$.

Bổ đề: Với $p_n$ là số nguyên tố thứ $n$ thì $p_n<n^2$ với $n>1$

Chứng minh:

Thấy $n=2,3,4,5$ luôn đúng. Xét $n\geq 6$

Áp dụng định lý PNT  ta có $\frac{p_n}{n}<log(n)+log(log(n))=log(nlog(n))<n\Rightarrow p_n<n^2$

Quay lại bài toán

 Có $n=1,2,3,4,5$ thỏa mãn. Xét $n\ge 6$ Có $p_n^2\geq 2^n$ thì cần  $n^4\geq 2^n$

Ta chứng minh với $n\geq 16$ thì $2^n\geq n^4$ bằng quy nạp. Thật vậy, giả sử khẳng định đúng với $n$, có $2^{n+1}\geq 2n^4\geq (n+1)^4$ ( do $n\geq 16$)

Vậy để $n$ thỏa mãn đề bài thì $1\leq n\leq 16$. Thử lại ta được $n=1,2,3,4,5,6,7,8.9$

 

P.s: một cách khác để chứng minh bổ đề trên là sử dụng giả thiết Andrica, nhưng có vẻ không quen thuộc bằng PNT.




#617220 Tìm tất cả $n$ để $p_{n} \ge (\sqrt{2...

Đã gửi bởi Chris yang on 27-02-2016 - 18:39 trong Số học

:-P Bạn có thể giới thiệu thêm về giả thiết Andrica luôn không nhỉ? Hình như $n = 9$ không đúng

Bài này có thể chặn thêm chặt nữa :-)  :wub: Mình cũng có lời giải khác

Giả thiết Andrica: Cho $p_i$ là số nguyên tố thứ $i$. Khi đó $\sqrt{p_{i+1}}-\sqrt{p_i}<1$. Về CM bạn có thể tham khảo Google =))

$p_9=23$. Luôn có $23^2>2^9$ nên $n=9$ đúng chứ nhỉ?

À, bạn post lời giải lên cho mọi người tham khảo với :)




#617333 Tìm tất cả $n$ để $p_{n} \ge (\sqrt{2...

Đã gửi bởi Chris yang on 28-02-2016 - 10:03 trong Số học

Nếu đã là giả thiết thì mình nghĩ chắc không thể vận dụng nó để chứng minh được, vì bản thân giả thiết còn chưa có chứng minh cho chính nó. :)

Mình nghĩ là được chứ, vì nó đã được chứng minh 

 

Lời giải của mình. Ta sẽ chứng minh kết quả sau đây: Với $n \ge 10$ thì $p_{n} < (\sqrt{2})^{n}$. Kiểm tra được $29 < (\sqrt{2})^{10} = 32$. Giả sử giả thiết đúng với $n \ge 10$, nghĩa là $p_{n} < (\sqrt{2})^{n}$.
Từ đó có $(\sqrt{2})^{n + 1} > \sqrt{2}.p_{n} > \frac{6}{5}p_{n}$. Để ý là $n \ge 10$ thì $p_{n} \ge 29$.
Định đề Nagura (kết quả mạnh hơn Bertrand): Với $n \ge 25$ thì giữa $n$ và $\frac{6n}{5}$ tồn tại một số nguyên tố.
Sử dụng định đề này cho ta kết quả $\frac{6}{5}p_{n} > p_{n + 1}$. Từ đó có điều phải chứng minh.
Thử lại từ $1$ đến $9$ cho ta nhận hết kết quả này :-) (như mình đã nói còn một số định đề chặn chặt hơn nữa (chặt hơn Nagura luôn) làm bài toán chặt hơn nữa)
p.s: To @ngocanh99: :-P Xin lỗi bạn, mình nhầm $p_{9} = 19$  :wub:

Cảm ơn bạn, trước giờ mình chưa hề biết đến kết quả này. Bữa làm bài này mình sử dụng Bertrand  nhưng không thành công  :ph34r: Bạn có thể cho mình xin ít tài liệu về định đề Nagura được không, mình search gg không có ra !




#638820 Tìm nghiệm tự nhiên của phương trình: $(x-1)!+1=x^k$

Đã gửi bởi Chris yang on 07-06-2016 - 22:53 trong Số học

Tìm nghiệm tự nhiên của phương trình: $(x-1)!+1=x^k$

Giải như sau:

 

Thử ta được $(x,k)=(2,1),(3,1),(5,2)$ thỏa mãn. Với $x>2$ dễ thấy $x$ lẻ

Xét $x>5$. Giả sử $x$ là hợp số thì bằng cách đặt $x=mn$ với $m,n\geq 2$ ta luôn chỉ ra được $x|(x-1)!$, kéo theo $x|1$ ( vô lý)

Vậy $x$ là số nguyên tố 

Theo khai triển nhị thức Newton thì $(x-1)!=[(x-1)+1]^k-1=\sum ^{k}_{i=1}\binom{k}{i}(x-1)^i \Rightarrow (x-2)!=k+\sum ^{k}_{i=2}\binom{k}{i}(x-1)^{i-1}\equiv k\pmod {x-1}$

Mặt khác, vì $x$ nguyên tố nên $x-1$ là hợp số. Tương tự cách trên ta chứng minh được $x-1|(x-2)!$, kéo theo $x-1|k\rightarrow x-1\leq k$

Mà hiển nhiên $k<x$ nên $k\leq x-1$, nếu không vế phải phương trình đã cho luôn lớn hơn vế trái phương trình đã cho. Do đó loại

 

Vậy $(x,k)=(2,1),(3,1), (5,2)$




#639578 Tìm nghiệm tự nhiên của phương trình: $(x-1)!+1=x^k$

Đã gửi bởi Chris yang on 11-06-2016 - 13:49 trong Số học

Chỗ $(****)$ mình không hiểu ? sao lại $x \leq x-1$

Mình viết nhầm, $k<x$ nên $k\leq x-1$ 




#569190 Tìm nghiệm nguyên: $3^{x}+4^{y}=7^{z}$

Đã gửi bởi Chris yang on 30-06-2015 - 23:02 trong Số học

Bài này vừa được đăng ở http://artofproblems..._integer_3x4y7z




#563588 Tìm nghiệm nguyên của pt $x^2+xy+y^2=x^2y^2$

Đã gửi bởi Chris yang on 04-06-2015 - 23:45 trong Số học

PT tương đương: $(x+y)^2=xy(xy+1)$

Mà $xy,xy+1$ là 2 số liên tiếp nên $xy=0\Leftrightarrow x=0$ hoặc $y=0$. Thay vào.....




#645760 Trung bình cộng của các ước của số $p^{m}q^{n...

Đã gửi bởi Chris yang on 21-07-2016 - 00:53 trong Số học

Cho $p,q$ là 2 số nguyên tố phân biệt. Chứng minh rằng: tồn tại các số nguyên dương $m, n$ sao cho trung bình cộng của các ước của số $p^{m}q^{n}$ là $1$ số nguyên.

ĐKĐB tương đương với tồn tại $m,n$ để $A=\frac{(p^{m+1}-1)(q^{n+1}-1)}{(p-1)(q-1)(m+1)(n+1)}=\frac{(p^m+p^{m-1}+...+1)(q^n+q^{n-1}+...+1)}{(m+1)(n+1)}\in \mathbb{Z}$

+) Nếu $p=2$ và $q$ lẻ:  chọn $n=q$ thì $n+1|q^n+q^{n-1}+...+1$, ta chọn $m+1=\frac{q^{n}+q^{n-1}+...+1}{n+1}=1+q+q^2+...+q^{n-1}$. Khi đó $A\in\mathbb{Z}$. TH $q=2$ và $p$ lẻ tương tự như vậy.

+) Nếu cả $p,q$ đều lẻ, chọn $m=p,n=q$ hiển nhiên ta thu luôn được $A\in\mathbb{Z}$ :D

Do đó luôn tồn tại $m,n\in\mathbb{Z}^+$ thỏa mãn điều kiện trên.




#493001 Toppic Các bài toán BĐT qua các kì thi olympic 30/4

Đã gửi bởi Chris yang on 14-04-2014 - 22:10 trong Bất đẳng thức - Cực trị

Mở rộng BĐT 30/4/2014

Cho $a,b,c$ là các số dương và k là một số dương bất kì. CMR

$\sum \frac{a}{\sqrt{ka^{2}+b^{2}+c^{2}}}\leq \sqrt{\frac{9}{k+2}}$

 

Theo em thì vẫn như cách làm bài trên, ta sẽ đi chứng minh

 

$\sum \frac{b^2+c^2}{ka^2+b^2+c^2}\geq \frac{6}{k+2}$

 

Sau đó áp dụng Bđt Cauchy- Schwarz thì ta có$\sum \frac{b^2+c^2}{ka^2+b^2+c^2}\geq \frac{4(a^2+b^2+c^2)^2}{\sum (b^2+c^2)(ka^2+b^2+c^2)}$

 

$=\frac{4(a^2+b^2+c^2)^2}{2(a^2+b^2+c^2)^2+(2k-2)(a^2b^2+b^2c^2+c^2a^2)}$

 

$\geq \frac{4(a^2+b^2+c^2)^2}{\frac{2(a^2+b^2+c^2)^2}{3(k+2)}}=\frac{6}{k+2}$

 

Bài toán đc cm xong :v




#580202 Topic post ảnh người yêu, bạn gái,...

Đã gửi bởi Chris yang on 09-08-2015 - 22:50 trong Góc giao lưu

Sau bao cảm xúc dồn nén bấy lâu ,tối nay em up ngay hình "Gấu" của em

attachicon.gif11219711_1492816341010230_8829147245020173718_n.jpg
 

Xinh gì dữ z trời  :like  :D Nhìn mê luôn rồi =))




#621113 Tiếp sức bất đẳng thức

Đã gửi bởi Chris yang on 19-03-2016 - 10:47 trong Bất đẳng thức và cực trị

bài bất mới 
Bài 95:cho a,b,c là các số thực dương

$ab^2+bc^2+ca^2=3$ 
chứng minh rằng $\frac{2a^5+3b^5}{ab}+\frac{2b^5+3c^5}{bc}+\frac{2c^5+3a^5}{ac} \geq 15(a^3+b^3+c^3-2)$ 
----------- Đề Thanh Hóa--- nguồn : facebook 
ps topic hay mong nó tiếp tục phát triển  >:)  >:)  >:)  >:)  >:)  >:)

Đã được giải tại đây

 

Em xin gửi hai bài ạ:

Bài 96: Cho $a,b,c,d$ là các số thực thỏa mãn điều kiện:

                                     $abc+bcd+cda+dab=a+b+c+d+\sqrt{2016}$.

Chứng minh rằng: $(a^2+1)(b^2+1)(c^2+1)(d^2+1)\geq 2016$.

Bài 97: Cho $a,b,c$ là các số thực dương thỏa mãn:

                                      $\frac{32}{a+32}+\frac{3}{2b+3}\leq \frac{4c}{4c+21}$.

Tìm giá trị nhỏ nhất của $abc$.

 

Bài 96:

Áp dụng BĐT Cauchy Schwarz

$(a^2+1)(b^2+1)(c^2+1)(d^2+1)=[(ab-1)^2+(a+b)^2][(c+d)^2+(cd-1)^2]$

hay $\text{VT}\geq [(ab-1)(c+d)+(cd-1)(a+b)]^2=2016$

 

Bài 97:

Điều kiện tương đương với $8abc\geq 256c^2+1344b+63a+384c+4032$

Áp dụng BĐT AM-GM: $\text{VP}=256c^2+384c+448b+448b+448b+21a+21a+21a+4032\geq 9\sqrt[9]{2^{39}3^67^6(abc)^3}$

$\Leftrightarrow 8abc\geq 9\sqrt[3]{2^{13}3^27^2abc}\Rightarrow a^2b^2c^2\geq 3^82^47^2\Rightarrow abc\geq 2268$

 

Bài 99: 

  Cho $a,b$ là hai số thực không âm thoả mãn $a+b=2$

  Chứng minh rằng: $2\leq \sqrt{a^{2}+b^{2}}+\sqrt{ab}\leq \sqrt{6}$

 

 

Vế đầu tiên:

 

Ta đi chứng minh $\sqrt{a^2+b^2}+\sqrt{ab}\geq \sqrt{a^2+b^2+2ab}=a+b$

BĐT trên tương đương với $2\sqrt{(a^2+b^2)ab}\geq ab$ ( luôn đúng theo AM-GM và với $a,b\geq 0$)

Suy ra $\sqrt{a^2+b^2}+\sqrt{ab}\geq 2$

 

Vế sau:

BĐT tương đương $\sqrt{4-2ab}\leq \sqrt{6}-\sqrt{ab}\Leftrightarrow 2+3ab\geq 2\sqrt{6ab}$ ( đúng theo BĐT AM-GM)

 

Bài toán được giải quyết!




#616555 Tiếp sức bất đẳng thức

Đã gửi bởi Chris yang on 23-02-2016 - 17:47 trong Bất đẳng thức và cực trị

Từ một bất đẳng thức phụ cơ bản, tớ đề xuất bài này:

Bài 85:

Với a,b,c là các số thực dương có tích là 8. Tìm giá trị lớn nhất của biểu thức:

$P= \frac{1}{2a+b+6}+\frac{1}{2b+c+6}+\frac{1}{2c+a+6}$

 

 tiện thể hỏi luôn ai biết topic nào có toán bằng tiếng anh ko ạ

Cho $(a,b,c) \mapsto (2a^2,2b^2,2c^2)$. Ta sẽ đưa bài toán về CM $P=\sum \frac{1}{4a^2+2b^2+6}\leq \frac{1}{4}$ với điều kiện $abc=1$

Theo BĐT BCS dạng cộng mẫu $2P\leq  \frac{1}{4}\sum \left ( \frac{1}{a^2+b^2+1}+\frac{1}{a^2+2} \right )=\frac{1}{4}\left ( \sum\frac{1}{a^2+b^2+1} +\sum \frac{1}{a^2+2} \right )$

Do đó, cần có $ \sum\frac{1}{a^2+b^2+1} +\sum \frac{1}{a^2+2} =\leq 2$

Thực hiện biến đổi ngược dấu, bài toán tương đương $\sum \frac{2(a^2+b^2)}{a^2+b^2+1}+\sum\frac{a^2}{a^2+2}=M+N\geq 5$ $(\star)$

Không mất TQ, giả sử $a\geq b\geq c$

Áp dụng BĐT Cauchy Schwarz: 

$M=\sum \frac{(a+b)^2}{a^2+b^2+1}+\sum \frac{(a-b)^2}{a^2+b^2+1}\geq \frac{4(a+b+c)^2+4(a-c)^2}{2(a^2+b^2+c^2)+3}\geq 4$

$\Leftrightarrow (a+b+c)^2+(a-c)^2\geq 2(a^2+b^2+c^2)+3\Leftrightarrow 2ab+2bc\geq b^2+3$

Có $(a-b)(b-c)\geq 0\rightarrow ab+bc\geq b^2+ac\rightarrow 2(ab+bc)\geq b^2+ac+ab+bc\geq b^2+3$ ( đúng vì $ab+bc+ac\geq 3$)

 

Do đó $M\geq 4$ $(1)$

Lại có $N=\sum \frac{a^2}{a^2+3}\geq \frac{(a+b+c)^2}{a^2+b^2+c^2+6}\geq 1\Leftrightarrow ab+bc+ac\geq 3$ ( đúng) $(2)$

Từ $(1)$ và $(2)$ suy ra $M+N\geq 5$, hay BĐT $(\star)$ đúng. Suy ra $P\leq \frac{1}{4}$

Dấu $=$ xảy ra khi ba biến bằng nhau

 

------------------------------------------------------------------------------------------------------------

Một số trang web toán bằng tiếng anh bạn có thể tham khảo là mathlinks.ro, math.stackexchange.com, brilliant.org




#616890 Tiếp sức bất đẳng thức

Đã gửi bởi Chris yang on 25-02-2016 - 18:33 trong Bất đẳng thức và cực trị

Bài 86

Cho $a,b,c>0$ thõa mãn $a+b+c=2$. C/m : $\sum \frac{bc}{20+3a^2} \le \frac{1}{16} $

Áp dụng BĐT Cauchy Schwarz:

$P=\sum \frac{bc}{3a^2+5(a+b+c)^2}\leq \sum \frac{bc}{2a(a+b+c)+\frac{14(a+b+c)^2}{3}}=\frac{3}{4}\sum \frac{bc}{10a+7b+7c}$

$P\leq \frac{3}{4.12^2}\sum \left ( \frac{5bc}{a+b}+\frac{5bc}{a+c}+\frac{2bc}{b+c} \right )=\frac{3}{4.12^2}\left ( 5(a+b+c)+\frac{2ab}{a+b}+\frac{2bc}{b+c}+\frac{2ac}{a+c} \right )\leq \frac{3.6}{4.12^2}(a+b+c)=\frac{1}{16}$

Dấu $=$ xảy ra khi ba biến bằng nhau

 

Bài 87:Cho $a,b,c>0$ và $a+b+c=3$.CMR:$(a+b)(b+c)(c+a)\geq (ab+c)(bc+a)(ca+b)$

Bài 88: Cho $a,b,c$ là các số dương.CMR:

$\frac{a^2}{2a^2+(b+c-a)^2}+\frac{b^2}{2b^2+(c+a-b)^2}+\frac{c^2}{2c^2+(a+b-c)^2}\leq 1$

 

Bài 87. Áp dụng BĐT AM GM ngược dấu:

$(ab+c)(bc+a)\leq \frac{(ab+c+bc+a)^2}{4}=\frac{(a+c)^2(b+1)^2}{4}$. Tương tự với các biểu thức còn lại, suy ra:

$VP^2\leq \frac{[(a+b)(b+c)(c+a)]^2(a+1)(b+1)(c+1)}{64}$. Áp dụng AM-GM lần nữa có $(a+1)(b+1)(c+1)\leq \left ( \frac{a+b+c+3}{3} \right )^3=8$

$\Rightarrow VP^2\leq (a+b)^2(b+c)^2(c+a)^2\Rightarrow (ab+c)(bc+a)(ca+b)\leq (a+b)(b+c)(c+a)$ ( đpcm)

 

Bài 88:

Đặt $(b+c-a,c+a-b,a+b-c)\mapsto (x,y,z)\Rightarrow (a,b,c)=(\frac{y+z}{2},\frac{x+z}{2}+\frac{x+y}{2}) $

Khi đó, thực hiện biến đổi ngược dấu,  BĐT cần chứng minh trở thành: $\sum \frac{x^2}{2x^2+(y+z)^2}\geq \frac{1}{2}$

BĐT này luôn đúng vì theo AM-GM ta có $\sum \frac{x^2}{2x^2+(y+z)^2}\geq \frac{x^2}{2x^2+2(y^2+z^2)}=\frac{1}{2}$ ( đpcm)




#614173 Tiếp sức bất đẳng thức

Đã gửi bởi Chris yang on 11-02-2016 - 15:48 trong Bất đẳng thức và cực trị

Bài 66:

Cho $ a, b, c $ các số thực không âm thỏa $ a+b+c=1 $.

Chứng minh rằng: $ \sqrt{4a+1}+\sqrt{4b+1}+\sqrt{4c+1} \ge \sqrt{5}+2 $

Ta có BĐT quen thuộc sau $\sqrt{4a+1}+\sqrt{4b+1}\geq \sqrt{4a+4b+1}+1$. Điều này $\Leftrightarrow 16ab\geq 0$ ( đúng)

Giờ ta đi CM $\sqrt{4a+4b+1}+\sqrt{4c+1}=\sqrt{5-4c}+\sqrt{4c+1}\geq \sqrt{5}+1\Leftrightarrow (5-4c)(4c+1)\geq 5\Leftrightarrow c(1-c)\geq 0$ ( luôn đúng do $c\leq 1$)

Ta có đpcm

Dấu $=$ xảy ra khi $(a,b,c)=(1,0,0)$ và hoán vị tương ứng

 

------------------------------------------------------------------------------------------------------------------

Bài 69: Cho $a,b,c>0$ thỏa mãn $a+b+c=\frac{1}{a}+\frac{1}{b}+\frac{1}{c}$

CMR $2(a+b+c)\geq \sqrt{a^2+3}+\sqrt{b^2+3}+\sqrt{c^2+3}$




#615225 Tiếp sức bất đẳng thức

Đã gửi bởi Chris yang on 15-02-2016 - 19:43 trong Bất đẳng thức và cực trị

 

 

 

 

Bài 18: Cho $a,b,c>0$ thỏa mãn $a\leq b\leq c$ và $a+b+c=\frac{1}{a}+\frac{1}{b}+\frac{1}{c}$.Tìm Min của $P=ab^2c^3$

 

 

Bài 18.

Ta có $abc(a+b+c)=ab+bc+ac\geq \sqrt{3abc(a+b+c)}\Rightarrow abc(a+b+c)\geq 3$

Từ giả thiết ta dễ dàng suy ra $ab+bc+ac\geq 3$. Do $a\leq b\leq c$ nên $bc\geq 1$

Khi đó $3bc^2\geq 3c\geq a+b+c$ $\Rightarrow P=ab^2c^3\geq \frac{abc(a+b+c)}{3}\geq 1$

Dấu $=$ xảy ra khi ba biến bằng nhau




#615325 Tiếp sức bất đẳng thức

Đã gửi bởi Chris yang on 16-02-2016 - 03:33 trong Bất đẳng thức và cực trị

một bài nữa:

Bài 79:

Với x,y,z là các số thực dương thỏa mãn điều kiện $x\sqrt{y}+y\sqrt{z}+z\sqrt{x}= 3$

Chứng minh rằng:

$P= \frac{x}{\sqrt{x+2y}}+\frac{y}{\sqrt{y+2z}}+\frac{z}{\sqrt{z+2x}}\geq \sqrt{xyz+2}$

 

Áp dụng BĐT Cauchy Schwarz:

 

$P=\sum \frac{x^2}{x\sqrt{x+2y}}\geq \frac{(x+y+z)^2}{\sum x\sqrt{x+2y}}\geq \frac{(x+y+z)^2}{\sqrt{(x+y+z)(x^2+y^2+z^2+2xy+2yz+2xz)}}=\sqrt{x+y+z}$

 

Ta sẽ chứng minh rằng $x+y+z\geq xyz+2$ $(\star)$

 

Thật vậy: Sử dụng AM-GM $(x+y+z)(\sqrt{x}+\sqrt{y}+\sqrt{z})=\sum (x\sqrt{x}+y\sqrt{x})+\sum x\sqrt{y}\geq 3\sum x\sqrt{y}=9$

 

Mà $(x+y+z)(\sqrt{x}+\sqrt{y}+\sqrt{z})\leq (x+y+z)\sqrt{3(x+y+z)}$ (AM-GM) nên $\Rightarrow x+y+z\geq 3$. Cũng dễ CMR $xyz\leq 1\rightarrow xyz+1\leq 3$

 

Do đó $(\star)$ đúng, ta có đpcm

 

Bài 80:

Cho $ x, y, z $  các số thực dương thỏa mãn $ x+y+z=3 $

Chứng minh rằng: $ x^{4}y^{4}z^{4}(x^{3}+y^{3}+z^{3}) \le 3 $

 

Viết lại $A=x^4y^4z^4(x^3+y^3+z^3)= x^4y^4z^4[(x+y+z)^3-3(x+y+z)(xy+yz+xz)+3xyz]$

 

$A=x^4y^4z^4[27-9(xy+yz+xz)+3xyz]$

 

Ta có BĐT phụ quen thuộc sau $(x+y+z)^5\geq 81xyz(x^2+y^2+z^2)\Leftrightarrow xyz[9-2(xy+yz+xz)]\leq 3$

 

$\Rightarrow 2A= x^3y^3z^3[54xyz-18xyz(xy+yz+xz)+6x^2y^2z^2]\leq x^3y^3z^3(27-27xyz+6x^2y^2z^2)=t^3(27-27t+6t^2)$

 

Ta cần chứng minh $t^3(27-27t+6t^2)\leq 6\Leftrightarrow 6(t^4+t^3+t^2+t+1)\geq 27t^3$. Điều này luôn đúng vì $t\leq 1$ theo AM-GM:

 

$6(t^4+t^2)+6t^3+6(t+1)\geq 12t^3+6t^3+12t^3>27t^3$

 

Do đó $2A\leq 6\rightarrow A\leq 3$ ( đpcm)




#614149 Tìm tất cả các số nguyên dương n sao cho (n-1)! chia hết cho $...

Đã gửi bởi Chris yang on 11-02-2016 - 13:51 trong Số học

Tìm tất cả các số nguyên dương n sao cho (n-1)! chia hết cho $n^{2}$

Mình nghĩ sẽ có vô số số $n$ thỏa mãn ( thử qua thấy nhiều lắm )

với $n$ nguyên tố thì hiển nhiên vô lý. 

Ta xét $n$ là hợp số có dạng $n=ab$ mà $a<b$

$(n-1)!=1.2.3...a(a+1)...b(b+1)...(ab-1)\vdots a^2b^2\Leftrightarrow (a+1)....(b-1)(b+1)...(ab-1)\vdots ab$

Bây giờ chỉ cần cho $\left\{\begin{matrix}2a<b+1\\ 2b<ab-1\end{matrix}\right.$ (luôn chọn đc với $a>2$).

Khi đó đoạn $a+1,....b-1$ sẽ chắc chắn chứa một thừa số chia hết cho $a$ và đoạn $b+1,...,ab-1$ chứa một thừa số chia hết cho $b$

Số $n$ khi phân tích thành tích hai thừa số $a,b$ thỏa mãn điều kiện trên đều thỏa mãn bài toán, nên có vô số số $n$